Proof by Induction: Explaining Step 3 to 4 | Math Homework

  • Thread starter Thread starter Robb
  • Start date Start date
  • Tags Tags
    Induction Proof
Click For Summary
In the discussion, a user seeks clarification on the disappearance of (-x(n+1)) between steps 3 and 4 of a proof by induction. The response highlights a minor error in the user's notes, specifically a misplaced parenthesis that affects the inequality's structure. The corrected version of step 2 is provided, emphasizing the correct factors involved. Additionally, it is noted that if the user has proven the Union Bound (Boole's Inequality), the result can be derived more easily, though careful consideration of sets and coin tossing is necessary. The conversation ultimately focuses on the importance of accurate notation in mathematical proofs.
Robb
Messages
225
Reaction score
8

Homework Statement


Attached are notes from class. Can someone please explain what happens to (-x(n+1)) in step 3 to step 4. Not sure why it goes away. Thanks!

Homework Equations

The Attempt at a Solution

 

Attachments

Physics news on Phys.org
Robb said:

Homework Statement


Attached are notes from class. Can someone please explain what happens to (-x(n+1)) in step 3 to step 4. Not sure why it goes away. Thanks!

Homework Equations

The Attempt at a Solution

Your notes (did you take them?) aren't very helpful, as there is a relatively minor error in them -- a misplaced parenthesis.
Here's the corrected version of step 2.
##\Pi_{i = 1}^{n + 1} (1 - x_i) \ge \left(1 - \sum_{i =1}^n x_i)\right)(1 - x_{n + 1})##
The right side of your inequality is incorrect, due to the parentheses being in the wrong place. One of the factors in the inequality is ##(1 - \sum_{i = 1}^n x_i)## and the other is ##(1 - x_{n + 1})##

The error is corrected in step 3. Now, to answer your question, what do ##-\sum_{i = 1}^n x_i## and ##-x_{n + 1}## combine to make?
 
Last edited:
the only things I'd add are

i.) if OP has proven Union Bound (Boole's Inequality), this result follows almost immediately -- though you need to think carefully about sets and coin tossing to get the result. - - - -
edit: geometric mean idea ran into too much trouble. Boole's Inequality still gives a very quick and satisfying answer.
 
Last edited:
Question: A clock's minute hand has length 4 and its hour hand has length 3. What is the distance between the tips at the moment when it is increasing most rapidly?(Putnam Exam Question) Answer: Making assumption that both the hands moves at constant angular velocities, the answer is ## \sqrt{7} .## But don't you think this assumption is somewhat doubtful and wrong?

Similar threads

  • · Replies 7 ·
Replies
7
Views
2K
  • · Replies 4 ·
Replies
4
Views
2K
  • · Replies 1 ·
Replies
1
Views
3K
Replies
5
Views
2K
  • · Replies 3 ·
Replies
3
Views
2K
Replies
1
Views
2K
  • · Replies 3 ·
Replies
3
Views
1K
  • · Replies 17 ·
Replies
17
Views
1K
  • · Replies 1 ·
Replies
1
Views
1K
  • · Replies 5 ·
Replies
5
Views
2K